LSAT and Law School Admissions Forum

Get expert LSAT preparation and law school admissions advice from PowerScore Test Preparation.

 Administrator
PowerScore Staff
  • PowerScore Staff
  • Posts: 8917
  • Joined: Feb 02, 2011
|
#26515
Complete Question Explanation

Weaken. The correct answer choice is (D)

This stimulus begins with the presentation of a common complaint: modifications to appliances are often made with no notice to consumers, so that new models are indistinguishable from older models. The author believes this complain to be unjustified, based on one premise:

  • Premise: Such modifications are invariably improvements.

    Conclusion: Therefore, consumers have little reason for such objections.

Only answer choice (D)effectively weakens this argument; if both new and old models are available at the same time, then consumers obviously want the newest, most improved model, and would have no way to distinguish the new from the old. This justifies the consumer complaint referenced by the dealer.

Answer choice (A) is irrelevant to the cited objection and therefore incorrect. Answer choice (B) strengthens the claim that model names don’t allow consumers to distinguish the older from the newer models. Answer choice (C) has no effect on the strength of the author’s conclusion. Although different model names might lead to different consumer objections, the relevant complaint here is that unchanged model names make new models indistinguishable from older models. Answer choice (E) provides insight into why manufacturers would opt not to change model names, but this is irrelevant to the dealer’s argument.
 kgalaraga93
  • Posts: 14
  • Joined: Aug 22, 2015
|
#19835
Hi There,

I am confused as to why the answer to this question is D and not A. I chose answer choice A because I felt that if appliances are purchased with the expectation that they will be used for several years, consumers cannot benefit from future models after the purchase of the first one. However the correct answer was choice D, which I also understand because If both improved and old versions are out the people who have the old versions cannot benefit from the new versions that are also out. Very similar reasoning to me which just adds to my confusion because they seem both right. Another question I have on top of that is in Weaken questions, is there only one answer choice that weakens the conclusion and every other choice either strengthens or does nothing? Any help with these questions would be greatly appreciated. Thank you!

Kim
 Robert Carroll
PowerScore Staff
  • PowerScore Staff
  • Posts: 1787
  • Joined: Dec 06, 2013
|
#19838
Kim,

The dealer's conclusion is that consumers have little reason to object to this practice - in other words, any possible confusion over what features you're getting with a certain model name shouldn't be objectionable for the consumers. The issue here is that people don't know what they're getting, and the dealer assumes (incorrectly) that people will always expect something with the same features as what they get or fewer - so they get exactly what they expect or are pleasantly surprised at some positive features they didn't expect. Since they might expect more and get less, the dealer's argument is weak.

The argument, its weakness, and the correct answer that exploits that weakness aren't about people who already have products - it's about people contemplating buying something that they don't have full information about, due to the practice of changing the models without changing the names. (D) gives a situation where people may expect better than what they get, and thus would be disappointed - so the dealer is wrong to say they have "little reason to object."

If the question asks you which answer choice weakens, you have one correct answer that weakens the argument and four incorrect answers that strengthen the argument or do nothing, like you said.

Robert Carroll
 blade21cn
  • Posts: 100
  • Joined: May 21, 2019
|
#65071
I've a hard time convincing myself how (C) weakens the argument. The dealer makes an incorrect assumption that consumers all want the old version, but may end up with a better, new version. But the reality is that consumers would prefer the new version and want to avoid getting the old version. It's already stated in the stimulus that "this practice makes it impossible for consumers to be certain that the appliance they are about to purchase is identical to ..." (C) just confirms a situation where this could happen. But the argument is not about whether it's actually hard to distinguish between the two versions. Rather, it's about assessing whether such given impossibility is a bad thing. How can an answer choice that only boosts/confirms the premise or background information strengthens or weakens the argument? Meanwhile, I fully understand the other answer choices are apparent losers. Thanks!
 Adam Tyson
PowerScore Staff
  • PowerScore Staff
  • Posts: 5153
  • Joined: Apr 14, 2011
|
#65183
Thanks for the question, blade21cn. Faced with the appliance dealer's claim that "consumers have little reason to object", your goal is to show that they DO have a reason to object. The author thinks that consumers shouldn't complain because even if they cannot tell the difference, they will be getting the better version, and answer C is suggesting that they might NOT get the better one, because the older, unimproved one is still on the floor. That gives consumers a reason to object - it's not just that the product may not be what they thought it was, but that it may lack an improvement that they thought they would get. Answer C does more than just repeat what the argument said - it confirms that what they complained about is, in fact, a valid complaint.

Putting this argument in simpler terms by paraphrasing:

Background statement: Some people are complaining about something
Premise: The thing they are complaining can only benefit them, never hurt them
Conclusion: They have no reason to complain

To weaken that, show that the thing they are complaining about CAN hurt them, giving them a reason to complain. Answer C does that.
 cmorris32
  • Posts: 92
  • Joined: May 05, 2020
|
#75502
Hi Powerscore!

I am having a difficult time understanding why D is the correct answer. I feel like it does not provide enough information to prove that releasing improved versions of appliances while selling the older versions actually results in customers wanting the new appliance. Just because improved versions of appliances typically become available before vendors stop selling older versions doesn't mean that customers are going to buy the new version. Can you please further explain this?

Thanks!
Caroline
 Adam Tyson
PowerScore Staff
  • PowerScore Staff
  • Posts: 5153
  • Joined: Apr 14, 2011
|
#76156
We don't need proof that this happens, Caroline - we just need a "reason to complain." Answer D sets up the possibility of confusion over what product you are getting. Do you want the improved version, or the earlier one? How can you tell what you are getting if they don't change the name? That possibility for confusion gives enough or a reason to complain that it weakens the claim that consumers have little reason to do so.

The correct answer does not have to disprove the conclusion, but just raise some doubts about it. Answer D raises at least some doubt, while none of the other answers do, and that makes it the best answer.

Get the most out of your LSAT Prep Plus subscription.

Analyze and track your performance with our Testing and Analytics Package.